choose the correct option regarding complete metric space












0












$begingroup$


let $X= mathbb{N}$,the set of postive integer consider the metrics $d_1,d_2 $ on $X$ given by
$d_1(m,n) =|m-n|$ , $m , n in X $



$d_2(m,n) =|frac{1}{m} - frac{1}{n}|$ , $m ,n in X $



let $X_1,X_2$ denotes the metric space $(X,d_1),(X,d_2)$ respectively .Then
choose the coorect option



$1.$ $X_1$ is complete



$2.$ $X_2$ is complete



$3.$ $X_1$ is totally bounded



$4.$ $X_2$ is totally bounded



My attempt : I got option $1)$ and option $3)$ beacuse the sequence $x_n = n in mathbb{N}$ is a Cauchy sequence for $d$. And $(x_n)$ don't converge to a limit.



is its true



Any hints/solution will be appreciated
thanks u










share|cite|improve this question











$endgroup$

















    0












    $begingroup$


    let $X= mathbb{N}$,the set of postive integer consider the metrics $d_1,d_2 $ on $X$ given by
    $d_1(m,n) =|m-n|$ , $m , n in X $



    $d_2(m,n) =|frac{1}{m} - frac{1}{n}|$ , $m ,n in X $



    let $X_1,X_2$ denotes the metric space $(X,d_1),(X,d_2)$ respectively .Then
    choose the coorect option



    $1.$ $X_1$ is complete



    $2.$ $X_2$ is complete



    $3.$ $X_1$ is totally bounded



    $4.$ $X_2$ is totally bounded



    My attempt : I got option $1)$ and option $3)$ beacuse the sequence $x_n = n in mathbb{N}$ is a Cauchy sequence for $d$. And $(x_n)$ don't converge to a limit.



    is its true



    Any hints/solution will be appreciated
    thanks u










    share|cite|improve this question











    $endgroup$















      0












      0








      0





      $begingroup$


      let $X= mathbb{N}$,the set of postive integer consider the metrics $d_1,d_2 $ on $X$ given by
      $d_1(m,n) =|m-n|$ , $m , n in X $



      $d_2(m,n) =|frac{1}{m} - frac{1}{n}|$ , $m ,n in X $



      let $X_1,X_2$ denotes the metric space $(X,d_1),(X,d_2)$ respectively .Then
      choose the coorect option



      $1.$ $X_1$ is complete



      $2.$ $X_2$ is complete



      $3.$ $X_1$ is totally bounded



      $4.$ $X_2$ is totally bounded



      My attempt : I got option $1)$ and option $3)$ beacuse the sequence $x_n = n in mathbb{N}$ is a Cauchy sequence for $d$. And $(x_n)$ don't converge to a limit.



      is its true



      Any hints/solution will be appreciated
      thanks u










      share|cite|improve this question











      $endgroup$




      let $X= mathbb{N}$,the set of postive integer consider the metrics $d_1,d_2 $ on $X$ given by
      $d_1(m,n) =|m-n|$ , $m , n in X $



      $d_2(m,n) =|frac{1}{m} - frac{1}{n}|$ , $m ,n in X $



      let $X_1,X_2$ denotes the metric space $(X,d_1),(X,d_2)$ respectively .Then
      choose the coorect option



      $1.$ $X_1$ is complete



      $2.$ $X_2$ is complete



      $3.$ $X_1$ is totally bounded



      $4.$ $X_2$ is totally bounded



      My attempt : I got option $1)$ and option $3)$ beacuse the sequence $x_n = n in mathbb{N}$ is a Cauchy sequence for $d$. And $(x_n)$ don't converge to a limit.



      is its true



      Any hints/solution will be appreciated
      thanks u







      general-topology






      share|cite|improve this question















      share|cite|improve this question













      share|cite|improve this question




      share|cite|improve this question








      edited Dec 17 '18 at 9:28







      jasmine

















      asked Dec 16 '18 at 19:28









      jasminejasmine

      1,884418




      1,884418






















          2 Answers
          2






          active

          oldest

          votes


















          2












          $begingroup$


          1. is OK, as Cauchy sequences are eventually constant.


          2. is false, indeed, as $x_n= n$ is a non-convergent Cauchy sequence.


          3. is false, as we cannot cover $X$ by finitely many balls of radius $1$.


          4. is true, as we can compactify it by adding a single point, e.g.







          share|cite|improve this answer









          $endgroup$





















            1












            $begingroup$

            For point $4.$, to explicitly see that $X_2$ is totally bounded, fix $epsilon > 0$. Choose $N > 3/epsilon.$ Then for any $M > N$, we see $$d_2(N,M) = left lvert frac 1 N - frac 1 M right rvert le frac 1 N + frac 1 M le 2epsilon/3 < epsilon.$$ Thus the set ${ m in mathbb N , : , m ge N}$ is contained in $B_epsilon(N)$. Thus the collection ${B_epsilon(n)}^N_{n=1}$ forms a finite cover of $mathbb N$ by balls of radius $epsilon$.



            The rest of the points can be elucidated with simple examples, as in the answer by @HennoBrandsma.






            share|cite|improve this answer









            $endgroup$













              Your Answer





              StackExchange.ifUsing("editor", function () {
              return StackExchange.using("mathjaxEditing", function () {
              StackExchange.MarkdownEditor.creationCallbacks.add(function (editor, postfix) {
              StackExchange.mathjaxEditing.prepareWmdForMathJax(editor, postfix, [["$", "$"], ["\\(","\\)"]]);
              });
              });
              }, "mathjax-editing");

              StackExchange.ready(function() {
              var channelOptions = {
              tags: "".split(" "),
              id: "69"
              };
              initTagRenderer("".split(" "), "".split(" "), channelOptions);

              StackExchange.using("externalEditor", function() {
              // Have to fire editor after snippets, if snippets enabled
              if (StackExchange.settings.snippets.snippetsEnabled) {
              StackExchange.using("snippets", function() {
              createEditor();
              });
              }
              else {
              createEditor();
              }
              });

              function createEditor() {
              StackExchange.prepareEditor({
              heartbeatType: 'answer',
              autoActivateHeartbeat: false,
              convertImagesToLinks: true,
              noModals: true,
              showLowRepImageUploadWarning: true,
              reputationToPostImages: 10,
              bindNavPrevention: true,
              postfix: "",
              imageUploader: {
              brandingHtml: "Powered by u003ca class="icon-imgur-white" href="https://imgur.com/"u003eu003c/au003e",
              contentPolicyHtml: "User contributions licensed under u003ca href="https://creativecommons.org/licenses/by-sa/3.0/"u003ecc by-sa 3.0 with attribution requiredu003c/au003e u003ca href="https://stackoverflow.com/legal/content-policy"u003e(content policy)u003c/au003e",
              allowUrls: true
              },
              noCode: true, onDemand: true,
              discardSelector: ".discard-answer"
              ,immediatelyShowMarkdownHelp:true
              });


              }
              });














              draft saved

              draft discarded


















              StackExchange.ready(
              function () {
              StackExchange.openid.initPostLogin('.new-post-login', 'https%3a%2f%2fmath.stackexchange.com%2fquestions%2f3043050%2fchoose-the-correct-option-regarding-complete-metric-space%23new-answer', 'question_page');
              }
              );

              Post as a guest















              Required, but never shown

























              2 Answers
              2






              active

              oldest

              votes








              2 Answers
              2






              active

              oldest

              votes









              active

              oldest

              votes






              active

              oldest

              votes









              2












              $begingroup$


              1. is OK, as Cauchy sequences are eventually constant.


              2. is false, indeed, as $x_n= n$ is a non-convergent Cauchy sequence.


              3. is false, as we cannot cover $X$ by finitely many balls of radius $1$.


              4. is true, as we can compactify it by adding a single point, e.g.







              share|cite|improve this answer









              $endgroup$


















                2












                $begingroup$


                1. is OK, as Cauchy sequences are eventually constant.


                2. is false, indeed, as $x_n= n$ is a non-convergent Cauchy sequence.


                3. is false, as we cannot cover $X$ by finitely many balls of radius $1$.


                4. is true, as we can compactify it by adding a single point, e.g.







                share|cite|improve this answer









                $endgroup$
















                  2












                  2








                  2





                  $begingroup$


                  1. is OK, as Cauchy sequences are eventually constant.


                  2. is false, indeed, as $x_n= n$ is a non-convergent Cauchy sequence.


                  3. is false, as we cannot cover $X$ by finitely many balls of radius $1$.


                  4. is true, as we can compactify it by adding a single point, e.g.







                  share|cite|improve this answer









                  $endgroup$




                  1. is OK, as Cauchy sequences are eventually constant.


                  2. is false, indeed, as $x_n= n$ is a non-convergent Cauchy sequence.


                  3. is false, as we cannot cover $X$ by finitely many balls of radius $1$.


                  4. is true, as we can compactify it by adding a single point, e.g.








                  share|cite|improve this answer












                  share|cite|improve this answer



                  share|cite|improve this answer










                  answered Dec 16 '18 at 22:16









                  Henno BrandsmaHenno Brandsma

                  112k348121




                  112k348121























                      1












                      $begingroup$

                      For point $4.$, to explicitly see that $X_2$ is totally bounded, fix $epsilon > 0$. Choose $N > 3/epsilon.$ Then for any $M > N$, we see $$d_2(N,M) = left lvert frac 1 N - frac 1 M right rvert le frac 1 N + frac 1 M le 2epsilon/3 < epsilon.$$ Thus the set ${ m in mathbb N , : , m ge N}$ is contained in $B_epsilon(N)$. Thus the collection ${B_epsilon(n)}^N_{n=1}$ forms a finite cover of $mathbb N$ by balls of radius $epsilon$.



                      The rest of the points can be elucidated with simple examples, as in the answer by @HennoBrandsma.






                      share|cite|improve this answer









                      $endgroup$


















                        1












                        $begingroup$

                        For point $4.$, to explicitly see that $X_2$ is totally bounded, fix $epsilon > 0$. Choose $N > 3/epsilon.$ Then for any $M > N$, we see $$d_2(N,M) = left lvert frac 1 N - frac 1 M right rvert le frac 1 N + frac 1 M le 2epsilon/3 < epsilon.$$ Thus the set ${ m in mathbb N , : , m ge N}$ is contained in $B_epsilon(N)$. Thus the collection ${B_epsilon(n)}^N_{n=1}$ forms a finite cover of $mathbb N$ by balls of radius $epsilon$.



                        The rest of the points can be elucidated with simple examples, as in the answer by @HennoBrandsma.






                        share|cite|improve this answer









                        $endgroup$
















                          1












                          1








                          1





                          $begingroup$

                          For point $4.$, to explicitly see that $X_2$ is totally bounded, fix $epsilon > 0$. Choose $N > 3/epsilon.$ Then for any $M > N$, we see $$d_2(N,M) = left lvert frac 1 N - frac 1 M right rvert le frac 1 N + frac 1 M le 2epsilon/3 < epsilon.$$ Thus the set ${ m in mathbb N , : , m ge N}$ is contained in $B_epsilon(N)$. Thus the collection ${B_epsilon(n)}^N_{n=1}$ forms a finite cover of $mathbb N$ by balls of radius $epsilon$.



                          The rest of the points can be elucidated with simple examples, as in the answer by @HennoBrandsma.






                          share|cite|improve this answer









                          $endgroup$



                          For point $4.$, to explicitly see that $X_2$ is totally bounded, fix $epsilon > 0$. Choose $N > 3/epsilon.$ Then for any $M > N$, we see $$d_2(N,M) = left lvert frac 1 N - frac 1 M right rvert le frac 1 N + frac 1 M le 2epsilon/3 < epsilon.$$ Thus the set ${ m in mathbb N , : , m ge N}$ is contained in $B_epsilon(N)$. Thus the collection ${B_epsilon(n)}^N_{n=1}$ forms a finite cover of $mathbb N$ by balls of radius $epsilon$.



                          The rest of the points can be elucidated with simple examples, as in the answer by @HennoBrandsma.







                          share|cite|improve this answer












                          share|cite|improve this answer



                          share|cite|improve this answer










                          answered Dec 16 '18 at 22:43









                          User8128User8128

                          10.8k1622




                          10.8k1622






























                              draft saved

                              draft discarded




















































                              Thanks for contributing an answer to Mathematics Stack Exchange!


                              • Please be sure to answer the question. Provide details and share your research!

                              But avoid



                              • Asking for help, clarification, or responding to other answers.

                              • Making statements based on opinion; back them up with references or personal experience.


                              Use MathJax to format equations. MathJax reference.


                              To learn more, see our tips on writing great answers.




                              draft saved


                              draft discarded














                              StackExchange.ready(
                              function () {
                              StackExchange.openid.initPostLogin('.new-post-login', 'https%3a%2f%2fmath.stackexchange.com%2fquestions%2f3043050%2fchoose-the-correct-option-regarding-complete-metric-space%23new-answer', 'question_page');
                              }
                              );

                              Post as a guest















                              Required, but never shown





















































                              Required, but never shown














                              Required, but never shown












                              Required, but never shown







                              Required, but never shown

































                              Required, but never shown














                              Required, but never shown












                              Required, but never shown







                              Required, but never shown







                              Popular posts from this blog

                              Plaza Victoria

                              In PowerPoint, is there a keyboard shortcut for bulleted / numbered list?

                              How to put 3 figures in Latex with 2 figures side by side and 1 below these side by side images but in...